Können Elementarteilchen auf einen kleineren Bereich als ihre Compton-Wellenlänge beschränkt werden?

Ich habe diese Frage gelesen:

Spagettifikation von Elementarteilchen (Elektron) und Nichtelementarteilchen (Proton).

und die Kommentare, wo es heißt:

Aber kein echtes Elementarteilchen kann auf einen Bereich beschränkt werden, der kleiner ist als seine Compton-Wellenlänge.

An diese Fragen:

Was ist die physikalische Bedeutung der Compton-Wellenlänge?

Einschließen eines Teilchens in einen Bereich, der kürzer ist als seine Compton-Wellenlänge

https://en.wikipedia.org/wiki/Compton_wavelength

Wo die Antwort von Veritas lautet:

Ja, das wird passieren. Aber man kann Teilchen nicht im Vakuum einschließen. Um ein Teilchen einzuschließen, müssen Sie ein gewisses Potenzial haben. Die Energie zur Bildung von Paaren muss genau aus diesem Bindungspotential stammen. Beispielsweise können Sie Elektronen mit einem sehr starken elektrischen Feld einschließen. Um ein Elektron in einem Bereich einzuschließen, der kleiner als seine Compton-Wellenlänge ist, benötigen Sie ein Feld mit genügend Energie, um Elektronenpositionspaare zu erzeugen. Partikel in einem Vakuum werden niemals eingeschlossen sein.

Welches ist also richtig?

Frage:

  1. Können Elementarteilchen auf einen kleineren Bereich als ihre Compton-Wellenlänge beschränkt werden?
Ich denke, die Antwort ist bereits aus den von Ihnen geposteten Links ableitbar. Der Versuch, ein Teilchen unterhalb seiner Compton-Wellenlänge einzuschließen, führt zu einer Paarbildung um es herum, wodurch die Bedeutung des Einschlusses eines einzelnen Teilchens aufgrund des Vorhandenseins einer Wolke neuer Teilchen um es herum im Wesentlichen zunichte gemacht wird.
@Avantgarde, woher wissen Sie, dass eine Paarbildung stattfinden wird, wenn wir nicht genau wissen, wie die Beschränkung erreicht wird? Und selbst wenn eine Paarbildung auftritt, wie verhindert dies, dass das ursprüngliche Teilchen in einem bekannten Bereich mit Dimensionen verschwindet, die kleiner als die Compton-Länge sind? Diese Art von Argumentation mit der Paarbildung scheint sehr dünn zu sein.
@JánLalinský Der Versuch, Entfernungen zu untersuchen, die kleiner als die Compton-Wellenlänge sind ( λ C ) impliziert, dass Sondierungsenergien größer sind als die Ruhemasse des beteiligten Teilchens. Dies ist das Regime, in dem die QFT (und damit die Partikelerzeugung) für die Beschreibung des Systems wesentlich wird. Außerdem habe ich nicht gesagt, dass das ursprüngliche Teilchen verschwinden wird: Untersuchungsabstände kleiner als λ C wird Energien übertragen Ruhemasse des ursprünglichen Teilchens.
@Avantgarde Sie scheinen eine hochenergetische Kollision zu beschreiben und denken, dass dies notwendig ist, um das Teilchen einzuschließen. Aber zu begrenzen bedeutet, die Position des Teilchens auf einen begrenzten verfügbaren Raum zu beschränken, nicht unbedingt, um es mit einem anderen Teilchen zu kollidieren. Einschluss kann erreicht werden, indem das Partikel an ein Atom oder molekulares System gebunden wird. Oder in einer statischen Feldfalle.
@JánLalinský Nein, das meine ich nicht. Sie können sich einschränken, wie Sie möchten. Danach wollen wir die Position des Teilchens messen. Reduzieren Sie nun die Unsicherheit der Positionsmessung unten λ C unbedingt bedeutet E M . Dies ist nur die Unschärferelation von Heisenberg.
@Avantgarde Nun, die Frage betrifft die Beschränkung, nicht die Messung der Position. Was ist E? Energie des Lichts, das verwendet wird, um das Teilchen zu betrachten?
@JánLalinský Das ist der Sinn von en.wikipedia.org/wiki/Measurement_in_quantum_mechanics . Erst nachdem Sie eine Messung an einem Partikel durchgeführt haben, um seine Position zu bestimmen, erhalten Sie Informationen darüber, inwieweit es „eingeschränkt“ ist. Wenn die Gefangenschaft perfekt war, dann Δ X = 0 und es gäbe keine Quantenmechanik. Die Messung der Position innerhalb λ C wird eine Impulsunsicherheit (des zu messenden Teilchens) induzieren, die so groß ist, dass a induziert wird Δ E genug für die Partikelproduktion. (Ich spreche hier grob, also ignorieren Δ , Faktoren von 2 , usw.)
@Avantgarde Ich denke, Sie ziehen voreilige Schlüsse. Die Beschränkung erfordert keine Messposition – siehe Antwort von Michale Momayezi. Und selbst wenn eine solche Messung durchgeführt wird, ist nicht klar, wie ein Mangel an Wissen über den Impuls die Paarbildung "induziert". Eine solche fehlende Kenntnis des Impulses ist immer vorhanden, auch wenn das Ergebnis der Positionsmessung eine größere Unsicherheit aufweist λ C , also sehe ich nicht ein, warum das Überschreiten dieser imaginären Grenze etwas ändern sollte.
@JánLalinský Confinement macht ohne Messung keinen objektiven Sinn , was ein entscheidender Aspekt der Quantenmechanik ist! Außerdem basiert die folgende Antwort auf dem Bohr-Modell, das voller Mängel ist, insbesondere: Es ist nicht relativistisch (wie es für die Begrenzung auf eine Entfernung erforderlich ist λ C ) und verstößt gegen die Heisenbergsche Unschärferelation. Sie können also das veraltete Bohr-Modell nicht verwenden, um die betreffenden Phänomene zu erklären.

Antworten (1)

Ich denke, es ist möglich, ein Elektron hauptsächlich auf einen Bereich zu beschränken, der kleiner ist als seine Compton-Wellenlänge ( λ C = 2.4 P M ).

Betrachten Sie zunächst Bohrs Modell für ein einzelnes Elektron im Feld eines Kerns der Ladung Ze. Der Grundzustand hat eine normalisierte Wahrscheinlichkeitsdichte proportional zu exp ( Z R / A 1 ) . Hier A 1 ist der Wasserstoff-Bohr-Radius von 53 pm. Dieser Bohr-Durchmesser ist also 44-mal λ C .

Betrachten Sie als nächstes einen vollständig ionisierten Urankern und fügen Sie ein Elektron hinzu. Sein Bohr-Radius wird 92-mal kleiner sein und daher A 92 = 0,58 P M . Der Durchmesser beträgt 13,06 Uhr, was immer noch erheblich kleiner ist als λ C .

Wenn wir ein zweites Elektron hinzufügen, um die s-Schale zu füllen, vergrößert sich das Orbital im Durchmesser. Zum Beispiel ist in Helium der Radius der vollen s-Schale 31pm statt des skalierten Bohr-Radius von 53pm/2; dh eine Steigerung von 17 %.

Wenn diese Näherung innerhalb eines Uranatoms gilt, können wir abschätzen, dass die s-Schale hauptsächlich auf eine Kugel kleiner als beschränkt ist λ C .

Dies kann auf schwerere Kerne ausgedehnt werden, solange sie stabil genug sind, um die Bildung einer Elektronenhülle zu ermöglichen, bevor sie zerfallen.

Ich denke nicht, dass es richtig ist, das Bohr-Modell zu verwenden, um Phänomene wie das in der Frage gestellte zu erklären. Das Modell hat viele Probleme, insbesondere weil es nichtrelativistisch ist und Heisenbergs Unbestimmtheitsprinzip verletzt.
@Avantgarde, ich stimme im Prinzip zu, aber in diesem Fall geben das Bohr-Modell, die nichtrelativistische Schrödinger-Gleichung und auch gängige relativistische Modelle auf der Grundlage der Dirac-Gleichung dieselbe Vorhersage für Wasserstoffatome; der typische Radius ist gegeben durch den Bohr-Radius dividiert durch Z . Wenn Z groß genug ist (größer als 43), der Radius ist kleiner als λ C / 2 . Die relativistischen Modelle sagen sogar etwas kleinere Radien voraus als die nicht-relativistischen.
@JánLalinský Es wäre gut, wenn Sie die entsprechenden Berechnungen und die Erklärung in eine Antwort einfügen könnten, damit jeder nachsehen kann. In dem Moment, in dem Sie sagen, dass sich ein Teilchen auf einem bestimmten Radius befindet, verwenden Sie effektiv alte quantentheoretische Konzepte (wie die des Bohr-Modells), die die Unfehlbarkeit der Teilchenposition annehmen. Dies ist in der modernen Quantentheorie verboten.
Das habe ich nicht gesagt; Ich sagte etwas über den typischen Radius, das ist der Wert R 0 In ψ = C e R / R 0 .
@JánLalinský Ja, ich weiß. Und solche Schlussfolgerungen über die radiale Position schließen die moderne Quantenmesstheorie im Kontext der Frage von OP aus. Mir ist kein Artikel/Buch bekannt, das typische Radien in QFT behandelt. Fühlen Sie sich jedoch frei, sie zu teilen, wenn Sie sie kennen.
@Avantgarde, die Frage musste im Rahmen der "modernen Quantenmesstheorie" oder QFT nicht beantwortet werden. Einschluss ruft Partikel in einer potentiellen Wand oder gebundene Zustände von zwei anziehenden Partikeln hervor; QFT hat wenig darüber zu sagen, die Gleichungen sind hartnäckig. Die meisten Berechnungen des gebundenen Zustands sind Näherungsmodelle, die auf der Dirac-Gleichung basieren. Also denke ich, dass Michaels Antwort gültig ist.
@JánLalinský Nein, wenn es um Längenskalen geht λ C , kommt man um QFT nicht herum, geschweige denn um das alte Bohr-Modell. Die Messung ist der Eckpfeiler der Quantenmechanik, ohne die Sie die (deterministische) klassische Mechanik ausführen werden, daher bin ich mir nicht sicher, warum Sie sie ignorieren möchten. Und sicherlich können gebundene Zustände in QFT gehandhabt werden, siehe physical.stackexchange.com/questions/217575/…